Let $Ain M_n(mathbb{Q})$ with $A^k=I_n$. If $j$ is a positive integer with $gcd(j,k)=1$, show that $...












3












$begingroup$



Let $Ain M_n(mathbb{Q})$ with $A^k=I_n$. If $j$ is a positive
integer with $gcd(j,k)=1$, show that $ operatorname{tr}(A)=
operatorname{tr}(A^j)$
.




I don't know how to start to prove that. I tried to find the matrix $B$ similar with $A$, but I am stuck...



Thank you.










share|cite|improve this question











$endgroup$



closed as off-topic by Saad, amWhy, Holo, Arnaud D., RRL Jan 10 at 17:28


This question appears to be off-topic. The users who voted to close gave this specific reason:


  • "This question is missing context or other details: Please provide additional context, which ideally explains why the question is relevant to you and our community. Some forms of context include: background and motivation, relevant definitions, source, possible strategies, your current progress, why the question is interesting or important, etc." – Saad, amWhy, Holo, Arnaud D., RRL

If this question can be reworded to fit the rules in the help center, please edit the question.





















    3












    $begingroup$



    Let $Ain M_n(mathbb{Q})$ with $A^k=I_n$. If $j$ is a positive
    integer with $gcd(j,k)=1$, show that $ operatorname{tr}(A)=
    operatorname{tr}(A^j)$
    .




    I don't know how to start to prove that. I tried to find the matrix $B$ similar with $A$, but I am stuck...



    Thank you.










    share|cite|improve this question











    $endgroup$



    closed as off-topic by Saad, amWhy, Holo, Arnaud D., RRL Jan 10 at 17:28


    This question appears to be off-topic. The users who voted to close gave this specific reason:


    • "This question is missing context or other details: Please provide additional context, which ideally explains why the question is relevant to you and our community. Some forms of context include: background and motivation, relevant definitions, source, possible strategies, your current progress, why the question is interesting or important, etc." – Saad, amWhy, Holo, Arnaud D., RRL

    If this question can be reworded to fit the rules in the help center, please edit the question.



















      3












      3








      3


      0



      $begingroup$



      Let $Ain M_n(mathbb{Q})$ with $A^k=I_n$. If $j$ is a positive
      integer with $gcd(j,k)=1$, show that $ operatorname{tr}(A)=
      operatorname{tr}(A^j)$
      .




      I don't know how to start to prove that. I tried to find the matrix $B$ similar with $A$, but I am stuck...



      Thank you.










      share|cite|improve this question











      $endgroup$





      Let $Ain M_n(mathbb{Q})$ with $A^k=I_n$. If $j$ is a positive
      integer with $gcd(j,k)=1$, show that $ operatorname{tr}(A)=
      operatorname{tr}(A^j)$
      .




      I don't know how to start to prove that. I tried to find the matrix $B$ similar with $A$, but I am stuck...



      Thank you.







      linear-algebra matrices trace






      share|cite|improve this question















      share|cite|improve this question













      share|cite|improve this question




      share|cite|improve this question








      edited Jan 10 at 16:32









      amWhy

      1




      1










      asked Jan 10 at 10:59









      w.sdkaw.sdka

      36919




      36919




      closed as off-topic by Saad, amWhy, Holo, Arnaud D., RRL Jan 10 at 17:28


      This question appears to be off-topic. The users who voted to close gave this specific reason:


      • "This question is missing context or other details: Please provide additional context, which ideally explains why the question is relevant to you and our community. Some forms of context include: background and motivation, relevant definitions, source, possible strategies, your current progress, why the question is interesting or important, etc." – Saad, amWhy, Holo, Arnaud D., RRL

      If this question can be reworded to fit the rules in the help center, please edit the question.







      closed as off-topic by Saad, amWhy, Holo, Arnaud D., RRL Jan 10 at 17:28


      This question appears to be off-topic. The users who voted to close gave this specific reason:


      • "This question is missing context or other details: Please provide additional context, which ideally explains why the question is relevant to you and our community. Some forms of context include: background and motivation, relevant definitions, source, possible strategies, your current progress, why the question is interesting or important, etc." – Saad, amWhy, Holo, Arnaud D., RRL

      If this question can be reworded to fit the rules in the help center, please edit the question.






















          2 Answers
          2






          active

          oldest

          votes


















          10












          $begingroup$

          Note that eigenvalues $lambda_i, i=1,ldots, n$ of $A$ are roots of the rational polynomial
          $$
          p(t) = det(tI-A)=prod_{i=1}^n (t-lambda_i).
          $$
          Since $Ax=lambda_i x$ implies $A^kx = x = lambda_i^k x$, we have ${lambda_i}subset mu_k ={zetainmathbb{C};|;zeta^k=1}$. Let $omega$ be the $k$-th primitive root of unity. Since $(j,k)=1$, we can define $sigmain text{Aut}(mathbb{Q}(mu_k)/mathbb{Q})$ by letting
          $$
          sigma(omega)= omega^j.
          $$
          Then, it holds
          $$
          p(t) = sigma(p(t)) = sigmaleft[prod_{i=1}^n (t-lambda_i)right]=prod_{i=1}^n (t-sigma(lambda_i))=prod_{i=1}^n (t-lambda_i^j).
          $$
          Now, it follows
          $$
          text{tr}(A) = sum_{i=1}^n lambda_i = sum_{i=1}^n lambda_i^j = text{tr}(A^j).
          $$






          share|cite|improve this answer











          $endgroup$









          • 1




            $begingroup$
            This is a nice answer that also nicely shows why we do need to do this over $mathbb{Q}$, since it will fail in suitable extensions.
            $endgroup$
            – Tobias Kildetoft
            Jan 10 at 11:46












          • $begingroup$
            I think this answer is wonderful..... I sincerely appreciate you!
            $endgroup$
            – w.sdka
            Jan 10 at 12:00










          • $begingroup$
            why does an eigenvalue $lambda_i$ occur with the same algebraic multiplicity as an eigenvalue $lambda_i^j$ of $A^j$?
            $endgroup$
            – M. Van
            Jan 10 at 13:19










          • $begingroup$
            @M.Van Because ${lambda_i} = {lambda_i^j}$ ..? Equality not as a set, but counted with multiplicity.
            $endgroup$
            – Song
            Jan 10 at 13:32












          • $begingroup$
            Umm... Could you explain why $lambda_i in mu_k$?.... why $prod_{i=1}^n lambda_i^k =1$ implies $lambda_i in mu_k$??
            $endgroup$
            – w.sdka
            Jan 10 at 13:43





















          1












          $begingroup$

          We must use the fact that $Ain M_n(mathbb{Q})$ because the result is false over $M_n(mathbb{C})$. Indeed, consider $A=diag(e^{2ipi/3},i)$ where $k=12$; then $trace(A)not= trace(A^5)$.



          $textbf{Proposition.}$ When $Ain M_n(mathbb{Q})$ and $(j,k)=1$, $spectrum(A)=spectrum(A^j)$.



          $textbf{Proof}.$
          We may assume that $order(A)=k$ ($k=min{l;A^l=I_2}$) and that $j<k$. Clearly, $order(A^j)=l$. $A$ is diagonalizable over $mathbb{C}$ and its minimal polynomial divides $x^k-1$.



          Therefore, $chi_A$, the characteristic polynomial of $A$, is a product of (irreducible) cyclotomic polynomials $phi_a(x)=Pi_{(u,a)=1}(x-e^{2ipi u/a})inmathbb{Q}[x]$ where $a|k$. Since $(j,a)=1$, it is easy to see that $phi_a(x)=Pi_{(u,a)=1}(x-(e^{2ipi u/a})^j)$ and we are done. $square$






          share|cite|improve this answer









          $endgroup$




















            2 Answers
            2






            active

            oldest

            votes








            2 Answers
            2






            active

            oldest

            votes









            active

            oldest

            votes






            active

            oldest

            votes









            10












            $begingroup$

            Note that eigenvalues $lambda_i, i=1,ldots, n$ of $A$ are roots of the rational polynomial
            $$
            p(t) = det(tI-A)=prod_{i=1}^n (t-lambda_i).
            $$
            Since $Ax=lambda_i x$ implies $A^kx = x = lambda_i^k x$, we have ${lambda_i}subset mu_k ={zetainmathbb{C};|;zeta^k=1}$. Let $omega$ be the $k$-th primitive root of unity. Since $(j,k)=1$, we can define $sigmain text{Aut}(mathbb{Q}(mu_k)/mathbb{Q})$ by letting
            $$
            sigma(omega)= omega^j.
            $$
            Then, it holds
            $$
            p(t) = sigma(p(t)) = sigmaleft[prod_{i=1}^n (t-lambda_i)right]=prod_{i=1}^n (t-sigma(lambda_i))=prod_{i=1}^n (t-lambda_i^j).
            $$
            Now, it follows
            $$
            text{tr}(A) = sum_{i=1}^n lambda_i = sum_{i=1}^n lambda_i^j = text{tr}(A^j).
            $$






            share|cite|improve this answer











            $endgroup$









            • 1




              $begingroup$
              This is a nice answer that also nicely shows why we do need to do this over $mathbb{Q}$, since it will fail in suitable extensions.
              $endgroup$
              – Tobias Kildetoft
              Jan 10 at 11:46












            • $begingroup$
              I think this answer is wonderful..... I sincerely appreciate you!
              $endgroup$
              – w.sdka
              Jan 10 at 12:00










            • $begingroup$
              why does an eigenvalue $lambda_i$ occur with the same algebraic multiplicity as an eigenvalue $lambda_i^j$ of $A^j$?
              $endgroup$
              – M. Van
              Jan 10 at 13:19










            • $begingroup$
              @M.Van Because ${lambda_i} = {lambda_i^j}$ ..? Equality not as a set, but counted with multiplicity.
              $endgroup$
              – Song
              Jan 10 at 13:32












            • $begingroup$
              Umm... Could you explain why $lambda_i in mu_k$?.... why $prod_{i=1}^n lambda_i^k =1$ implies $lambda_i in mu_k$??
              $endgroup$
              – w.sdka
              Jan 10 at 13:43


















            10












            $begingroup$

            Note that eigenvalues $lambda_i, i=1,ldots, n$ of $A$ are roots of the rational polynomial
            $$
            p(t) = det(tI-A)=prod_{i=1}^n (t-lambda_i).
            $$
            Since $Ax=lambda_i x$ implies $A^kx = x = lambda_i^k x$, we have ${lambda_i}subset mu_k ={zetainmathbb{C};|;zeta^k=1}$. Let $omega$ be the $k$-th primitive root of unity. Since $(j,k)=1$, we can define $sigmain text{Aut}(mathbb{Q}(mu_k)/mathbb{Q})$ by letting
            $$
            sigma(omega)= omega^j.
            $$
            Then, it holds
            $$
            p(t) = sigma(p(t)) = sigmaleft[prod_{i=1}^n (t-lambda_i)right]=prod_{i=1}^n (t-sigma(lambda_i))=prod_{i=1}^n (t-lambda_i^j).
            $$
            Now, it follows
            $$
            text{tr}(A) = sum_{i=1}^n lambda_i = sum_{i=1}^n lambda_i^j = text{tr}(A^j).
            $$






            share|cite|improve this answer











            $endgroup$









            • 1




              $begingroup$
              This is a nice answer that also nicely shows why we do need to do this over $mathbb{Q}$, since it will fail in suitable extensions.
              $endgroup$
              – Tobias Kildetoft
              Jan 10 at 11:46












            • $begingroup$
              I think this answer is wonderful..... I sincerely appreciate you!
              $endgroup$
              – w.sdka
              Jan 10 at 12:00










            • $begingroup$
              why does an eigenvalue $lambda_i$ occur with the same algebraic multiplicity as an eigenvalue $lambda_i^j$ of $A^j$?
              $endgroup$
              – M. Van
              Jan 10 at 13:19










            • $begingroup$
              @M.Van Because ${lambda_i} = {lambda_i^j}$ ..? Equality not as a set, but counted with multiplicity.
              $endgroup$
              – Song
              Jan 10 at 13:32












            • $begingroup$
              Umm... Could you explain why $lambda_i in mu_k$?.... why $prod_{i=1}^n lambda_i^k =1$ implies $lambda_i in mu_k$??
              $endgroup$
              – w.sdka
              Jan 10 at 13:43
















            10












            10








            10





            $begingroup$

            Note that eigenvalues $lambda_i, i=1,ldots, n$ of $A$ are roots of the rational polynomial
            $$
            p(t) = det(tI-A)=prod_{i=1}^n (t-lambda_i).
            $$
            Since $Ax=lambda_i x$ implies $A^kx = x = lambda_i^k x$, we have ${lambda_i}subset mu_k ={zetainmathbb{C};|;zeta^k=1}$. Let $omega$ be the $k$-th primitive root of unity. Since $(j,k)=1$, we can define $sigmain text{Aut}(mathbb{Q}(mu_k)/mathbb{Q})$ by letting
            $$
            sigma(omega)= omega^j.
            $$
            Then, it holds
            $$
            p(t) = sigma(p(t)) = sigmaleft[prod_{i=1}^n (t-lambda_i)right]=prod_{i=1}^n (t-sigma(lambda_i))=prod_{i=1}^n (t-lambda_i^j).
            $$
            Now, it follows
            $$
            text{tr}(A) = sum_{i=1}^n lambda_i = sum_{i=1}^n lambda_i^j = text{tr}(A^j).
            $$






            share|cite|improve this answer











            $endgroup$



            Note that eigenvalues $lambda_i, i=1,ldots, n$ of $A$ are roots of the rational polynomial
            $$
            p(t) = det(tI-A)=prod_{i=1}^n (t-lambda_i).
            $$
            Since $Ax=lambda_i x$ implies $A^kx = x = lambda_i^k x$, we have ${lambda_i}subset mu_k ={zetainmathbb{C};|;zeta^k=1}$. Let $omega$ be the $k$-th primitive root of unity. Since $(j,k)=1$, we can define $sigmain text{Aut}(mathbb{Q}(mu_k)/mathbb{Q})$ by letting
            $$
            sigma(omega)= omega^j.
            $$
            Then, it holds
            $$
            p(t) = sigma(p(t)) = sigmaleft[prod_{i=1}^n (t-lambda_i)right]=prod_{i=1}^n (t-sigma(lambda_i))=prod_{i=1}^n (t-lambda_i^j).
            $$
            Now, it follows
            $$
            text{tr}(A) = sum_{i=1}^n lambda_i = sum_{i=1}^n lambda_i^j = text{tr}(A^j).
            $$







            share|cite|improve this answer














            share|cite|improve this answer



            share|cite|improve this answer








            edited Jan 10 at 17:34

























            answered Jan 10 at 11:40









            SongSong

            11.7k628




            11.7k628








            • 1




              $begingroup$
              This is a nice answer that also nicely shows why we do need to do this over $mathbb{Q}$, since it will fail in suitable extensions.
              $endgroup$
              – Tobias Kildetoft
              Jan 10 at 11:46












            • $begingroup$
              I think this answer is wonderful..... I sincerely appreciate you!
              $endgroup$
              – w.sdka
              Jan 10 at 12:00










            • $begingroup$
              why does an eigenvalue $lambda_i$ occur with the same algebraic multiplicity as an eigenvalue $lambda_i^j$ of $A^j$?
              $endgroup$
              – M. Van
              Jan 10 at 13:19










            • $begingroup$
              @M.Van Because ${lambda_i} = {lambda_i^j}$ ..? Equality not as a set, but counted with multiplicity.
              $endgroup$
              – Song
              Jan 10 at 13:32












            • $begingroup$
              Umm... Could you explain why $lambda_i in mu_k$?.... why $prod_{i=1}^n lambda_i^k =1$ implies $lambda_i in mu_k$??
              $endgroup$
              – w.sdka
              Jan 10 at 13:43
















            • 1




              $begingroup$
              This is a nice answer that also nicely shows why we do need to do this over $mathbb{Q}$, since it will fail in suitable extensions.
              $endgroup$
              – Tobias Kildetoft
              Jan 10 at 11:46












            • $begingroup$
              I think this answer is wonderful..... I sincerely appreciate you!
              $endgroup$
              – w.sdka
              Jan 10 at 12:00










            • $begingroup$
              why does an eigenvalue $lambda_i$ occur with the same algebraic multiplicity as an eigenvalue $lambda_i^j$ of $A^j$?
              $endgroup$
              – M. Van
              Jan 10 at 13:19










            • $begingroup$
              @M.Van Because ${lambda_i} = {lambda_i^j}$ ..? Equality not as a set, but counted with multiplicity.
              $endgroup$
              – Song
              Jan 10 at 13:32












            • $begingroup$
              Umm... Could you explain why $lambda_i in mu_k$?.... why $prod_{i=1}^n lambda_i^k =1$ implies $lambda_i in mu_k$??
              $endgroup$
              – w.sdka
              Jan 10 at 13:43










            1




            1




            $begingroup$
            This is a nice answer that also nicely shows why we do need to do this over $mathbb{Q}$, since it will fail in suitable extensions.
            $endgroup$
            – Tobias Kildetoft
            Jan 10 at 11:46






            $begingroup$
            This is a nice answer that also nicely shows why we do need to do this over $mathbb{Q}$, since it will fail in suitable extensions.
            $endgroup$
            – Tobias Kildetoft
            Jan 10 at 11:46














            $begingroup$
            I think this answer is wonderful..... I sincerely appreciate you!
            $endgroup$
            – w.sdka
            Jan 10 at 12:00




            $begingroup$
            I think this answer is wonderful..... I sincerely appreciate you!
            $endgroup$
            – w.sdka
            Jan 10 at 12:00












            $begingroup$
            why does an eigenvalue $lambda_i$ occur with the same algebraic multiplicity as an eigenvalue $lambda_i^j$ of $A^j$?
            $endgroup$
            – M. Van
            Jan 10 at 13:19




            $begingroup$
            why does an eigenvalue $lambda_i$ occur with the same algebraic multiplicity as an eigenvalue $lambda_i^j$ of $A^j$?
            $endgroup$
            – M. Van
            Jan 10 at 13:19












            $begingroup$
            @M.Van Because ${lambda_i} = {lambda_i^j}$ ..? Equality not as a set, but counted with multiplicity.
            $endgroup$
            – Song
            Jan 10 at 13:32






            $begingroup$
            @M.Van Because ${lambda_i} = {lambda_i^j}$ ..? Equality not as a set, but counted with multiplicity.
            $endgroup$
            – Song
            Jan 10 at 13:32














            $begingroup$
            Umm... Could you explain why $lambda_i in mu_k$?.... why $prod_{i=1}^n lambda_i^k =1$ implies $lambda_i in mu_k$??
            $endgroup$
            – w.sdka
            Jan 10 at 13:43






            $begingroup$
            Umm... Could you explain why $lambda_i in mu_k$?.... why $prod_{i=1}^n lambda_i^k =1$ implies $lambda_i in mu_k$??
            $endgroup$
            – w.sdka
            Jan 10 at 13:43













            1












            $begingroup$

            We must use the fact that $Ain M_n(mathbb{Q})$ because the result is false over $M_n(mathbb{C})$. Indeed, consider $A=diag(e^{2ipi/3},i)$ where $k=12$; then $trace(A)not= trace(A^5)$.



            $textbf{Proposition.}$ When $Ain M_n(mathbb{Q})$ and $(j,k)=1$, $spectrum(A)=spectrum(A^j)$.



            $textbf{Proof}.$
            We may assume that $order(A)=k$ ($k=min{l;A^l=I_2}$) and that $j<k$. Clearly, $order(A^j)=l$. $A$ is diagonalizable over $mathbb{C}$ and its minimal polynomial divides $x^k-1$.



            Therefore, $chi_A$, the characteristic polynomial of $A$, is a product of (irreducible) cyclotomic polynomials $phi_a(x)=Pi_{(u,a)=1}(x-e^{2ipi u/a})inmathbb{Q}[x]$ where $a|k$. Since $(j,a)=1$, it is easy to see that $phi_a(x)=Pi_{(u,a)=1}(x-(e^{2ipi u/a})^j)$ and we are done. $square$






            share|cite|improve this answer









            $endgroup$


















              1












              $begingroup$

              We must use the fact that $Ain M_n(mathbb{Q})$ because the result is false over $M_n(mathbb{C})$. Indeed, consider $A=diag(e^{2ipi/3},i)$ where $k=12$; then $trace(A)not= trace(A^5)$.



              $textbf{Proposition.}$ When $Ain M_n(mathbb{Q})$ and $(j,k)=1$, $spectrum(A)=spectrum(A^j)$.



              $textbf{Proof}.$
              We may assume that $order(A)=k$ ($k=min{l;A^l=I_2}$) and that $j<k$. Clearly, $order(A^j)=l$. $A$ is diagonalizable over $mathbb{C}$ and its minimal polynomial divides $x^k-1$.



              Therefore, $chi_A$, the characteristic polynomial of $A$, is a product of (irreducible) cyclotomic polynomials $phi_a(x)=Pi_{(u,a)=1}(x-e^{2ipi u/a})inmathbb{Q}[x]$ where $a|k$. Since $(j,a)=1$, it is easy to see that $phi_a(x)=Pi_{(u,a)=1}(x-(e^{2ipi u/a})^j)$ and we are done. $square$






              share|cite|improve this answer









              $endgroup$
















                1












                1








                1





                $begingroup$

                We must use the fact that $Ain M_n(mathbb{Q})$ because the result is false over $M_n(mathbb{C})$. Indeed, consider $A=diag(e^{2ipi/3},i)$ where $k=12$; then $trace(A)not= trace(A^5)$.



                $textbf{Proposition.}$ When $Ain M_n(mathbb{Q})$ and $(j,k)=1$, $spectrum(A)=spectrum(A^j)$.



                $textbf{Proof}.$
                We may assume that $order(A)=k$ ($k=min{l;A^l=I_2}$) and that $j<k$. Clearly, $order(A^j)=l$. $A$ is diagonalizable over $mathbb{C}$ and its minimal polynomial divides $x^k-1$.



                Therefore, $chi_A$, the characteristic polynomial of $A$, is a product of (irreducible) cyclotomic polynomials $phi_a(x)=Pi_{(u,a)=1}(x-e^{2ipi u/a})inmathbb{Q}[x]$ where $a|k$. Since $(j,a)=1$, it is easy to see that $phi_a(x)=Pi_{(u,a)=1}(x-(e^{2ipi u/a})^j)$ and we are done. $square$






                share|cite|improve this answer









                $endgroup$



                We must use the fact that $Ain M_n(mathbb{Q})$ because the result is false over $M_n(mathbb{C})$. Indeed, consider $A=diag(e^{2ipi/3},i)$ where $k=12$; then $trace(A)not= trace(A^5)$.



                $textbf{Proposition.}$ When $Ain M_n(mathbb{Q})$ and $(j,k)=1$, $spectrum(A)=spectrum(A^j)$.



                $textbf{Proof}.$
                We may assume that $order(A)=k$ ($k=min{l;A^l=I_2}$) and that $j<k$. Clearly, $order(A^j)=l$. $A$ is diagonalizable over $mathbb{C}$ and its minimal polynomial divides $x^k-1$.



                Therefore, $chi_A$, the characteristic polynomial of $A$, is a product of (irreducible) cyclotomic polynomials $phi_a(x)=Pi_{(u,a)=1}(x-e^{2ipi u/a})inmathbb{Q}[x]$ where $a|k$. Since $(j,a)=1$, it is easy to see that $phi_a(x)=Pi_{(u,a)=1}(x-(e^{2ipi u/a})^j)$ and we are done. $square$







                share|cite|improve this answer












                share|cite|improve this answer



                share|cite|improve this answer










                answered Jan 10 at 12:38









                loup blancloup blanc

                23.1k21850




                23.1k21850















                    Popular posts from this blog

                    Can a sorcerer learn a 5th-level spell early by creating spell slots using the Font of Magic feature?

                    Does disintegrating a polymorphed enemy still kill it after the 2018 errata?

                    A Topological Invariant for $pi_3(U(n))$